Home / ECE / Measurements and Instrumentation :: Discussion

Discussion :: Measurements and Instrumentation

  1. If an inductance L is connected in one arm of bridge and resistance R1, R2, R3 in other three arms

  2. A.
    the bridge cannot be balanced
    B.
    the bridge can be balanced
    C.
    the bridge is balanced for some specified value of frequency
    D.
    the bridge is balanced for some specific values of R1, R2, R3

    View Answer

    Workspace

    Answer : Option A

    Explanation :

    To balance the bridge both magnitudes and phase angles of voltages have to be balanced.


Be The First To Comment